Đến nội dung

dogsteven nội dung

Có 1000 mục bởi dogsteven (Tìm giới hạn từ 28-04-2020)



Sắp theo                Sắp xếp  

#540422 Topic về Bất đẳng thức, cực trị THCS

Đã gửi bởi dogsteven on 11-01-2015 - 19:26 trong Bất đẳng thức và cực trị

Cho a>0,b>0,và a+b$\leq$1.Tìm GTNN của biểu thức A=$a^{2}$+$b^{2}+\frac{1}{a^{2}}+\frac{1}{b^{2}}$

Đề thi Sóc Trăng năm nay phải không nhỉ.

 

$a^2+b^2+\dfrac{1}{a^2}+\dfrac{1}{b^2}\geqslant \dfrac{(a+b)^2}{2}+\dfrac{8}{(a+b)^2}=\dfrac{(a+b)^2}{2}+\dfrac{1}{2(a+b)^2}+\dfrac{15}{2(a+b)^2}\geqslant 2\sqrt{\dfrac{(a+b)^2}{4(a+b)^2}}+\dfrac{15}{2}=\dfrac{17}{2}$




#537448 Topic về Bất đẳng thức, cực trị THCS

Đã gửi bởi dogsteven on 12-12-2014 - 16:44 trong Bất đẳng thức và cực trị

cho a,b,c$\in \mathbb{R}$ thỏa mãn ab+bc+ca=3. Tìm min 

P=$\frac{a+3}{b^{2}+1}+\frac{b+3}{c^{2}+1}+\frac{c+3}{a^{2}+1}$

$$\dfrac{a+3}{b^2+1}=a+3-\dfrac{b^2(a+3)}{b^2+1} \geqslant a+3-\dfrac{ab+3b}{2}$$

Tương tự.




#544322 Topic về Bất đẳng thức, cực trị THCS

Đã gửi bởi dogsteven on 15-02-2015 - 19:09 trong Bất đẳng thức và cực trị

Với $x, y\geq 1$.

Chứng minh:

$\frac{1}{1+x^{2}}+\frac{1}{1+y^{2}}\geq \frac{2}{1+xy}$

Cách 1. Quy đồng lên biến đổi tương đương ra $(xy-1)(x-y)^2\geqslant 0$ luôn đúng.

 

Cách 2. Cách này đẹp hơn.

Xét hàm số $f(x)=\dfrac{1}{e^x+1}$ trên $\mathbb{R}^{+}$. Khi đó $f''(x)=\dfrac{e^x(e^x-1)}{(e^x+1)^3} \geqslant 0$

Áp dụng bất đẳng thức Jensen: $f(\ln x^2)+f(\ln y^2)\geqslant 2f\left(\dfrac{\ln x^2+ \ln y^2}{2}\right)=2f(\ln xy)$ hay $\dfrac{1}{x^2+1}+\dfrac{1}{y^2+1}\geqslant \dfrac{2}{xy+1}$




#540915 Topic về Bất đẳng thức, cực trị THCS

Đã gửi bởi dogsteven on 15-01-2015 - 16:14 trong Bất đẳng thức và cực trị

Mình nghĩ phần $(\sqrt{x}+\sqrt{y})(\sqrt{x}+\sqrt{z})\geq (\sqrt{x}.\sqrt{x}+\sqrt{y}.\sqrt{z})$ là theo bất đẳng thức Bu-nhi-a-cốp-xki mới đúng chứ !
Cám ơn bạn rất nhiều !Cách giải rất dễ hiểu

Cauchy-Schwarz với Bunyakovsky là một.




#553171 Topic luyện thi vào lớp 10 năm 2013 – 2014 (Hình học)

Đã gửi bởi dogsteven on 11-04-2015 - 14:29 trong Hình học

Bài 1. Hướng làm.

Dựng đường thẳng qua $B$  song song với $OO'$ cắt $(O), (O')$ lần lược tại $C', D'$

Dùng tính chất độ hình chiếu một đoạn thẳng không vượt quá đoạn thẳng đó.




#553174 Topic luyện thi vào lớp 10 năm 2013 – 2014 (Hình học)

Đã gửi bởi dogsteven on 11-04-2015 - 14:41 trong Hình học

Giới thiệu một số bài bất đẳng thức và cực trị hình học 9

 

Bài 1: Hai đường tròn (O) và (O’) cắt nhau tại A, B. Một đường thẳng qua A cắt các đường tròn đó lần lượt tại C và D nằm khác phía đối với AB (C thuộc đường tròn tâm O). Chứng minh rằng $CD\leq 2.OO'$

Bài 2: Cho đường tròn (O) và điểm A cố định nằm ngoài đường tròn. Xét đường kính BC của (O). Tìm vị trí của đường kính BC để AB + AC nhỏ nhất

Bài 2. Xét $A, B,C$ thẳng hàng thì ....

Xét $A,B,C$ không thẳng hàng thì dựng hình bình hành $A'BAC$




#526367 $\boxed{\text{Chuyên Đề}}$ Bất đẳng thức - Cực trị

Đã gửi bởi dogsteven on 27-09-2014 - 10:35 trong Bất đẳng thức và cực trị

Tớ chỉ tính toán sai thôi mà!Mà đề bài cho $a,b,c>0$ có dấu bằng à bạn.Nếu có chỉ giáo xem

 

 

Thì ai nói có dấu bằng đâu.




#526239 $\boxed{\text{Chuyên Đề}}$ Bất đẳng thức - Cực trị

Đã gửi bởi dogsteven on 26-09-2014 - 12:14 trong Bất đẳng thức và cực trị

Cho a,b,c dương, abc=1.CMR:

   $\frac{1}{1+a+b}+\frac{1}{1+b+c}+\frac{1}{1+a+c}\leq 1$

 

Đặt $(a;b;c)=(x^2;y^2;z^2)$ với $x,y,z >0$, khi đó $xyz=1$

 

Tương đương với: $\sum \dfrac{x^2+y^2}{x^2+y^2+1} \geqslant 2$

 

$\Leftrightarrow \sum \dfrac{(x+y)^2}{x^2+y^2+1}+\sum \dfrac{(x-y)^2}{x^2+y^2+1} \geqslant 4$

 

Theo BDT Cauchy-Schwarz:

$\sum \dfrac{(x+y)^2}{x^2+y^2+1} \geqslant \dfrac{4(x+y+z)^2}{2x^2+2y^2+2z^2+3}$

$\sum \dfrac{(x-y)^2}{x^2+y^2+1} \geqslant \dfrac{4(x-z)^2}{2x^2+2y^2+2z^2+3}$

Theo BDT Cauchy: $xy+yz+zx \geqslant 3$

 

Ta cần chứng minh: $(x+y+z)^2+(x-z)^2 \geqslant 2x^2+2y^2+2z^2+xy+yz+zx$

 

$\Leftrightarrow xy+yz-y^2-zx \geqslant 0 \Leftrightarrow (x-y)(y-z) \geqslant 0$

 

Tương tự ta cũng có $(y-z)(z-x) \geqslant 0$ và $(z-x)(x-y) \geqslant 0$

 

Chỉ cần 1 BDT trong 3 BDT trên đúng là bài toán được chứng minh.

 

Điều này hiển nhiên vì $[(x-y)(y-z)(z-x)]^2 \geqslant 0$

 

Đẳng thức xảy ra khi $x=y=z=1$

 

Có vẻ hơi dài :D




#526359 $\boxed{\text{Chuyên Đề}}$ Bất đẳng thức - Cực trị

Đã gửi bởi dogsteven on 27-09-2014 - 09:17 trong Bất đẳng thức và cực trị

Ta có:BĐT pcm<=>$(4a+b+c)(4b+a+c)(4c+a+b)> 25(a+b)(b+c)(c+a)$

<=>$a^3+b^3+c^3+7abc>ab(a+b)+bc(b+c)+ac(a+c)$

BĐT trên đúng vì $a^3+b^3+c^3\geq ab(a+b)+bc(b+c)+ac(a+c)$ là bđt schur

$a,b,c>0$=>$7abc>0$ => đpcm

 

Sai rồi, BDT Schur bậc 3 là $a^3+b^3+c^3+3abc \geqslant ab(a+b)+bc(b+c)+ca(c+a)$

 

Chứng minh:

 

$f(a;b;c)=a^3+b^3+c^3+3abc-a^2(b+c)-b^2(c+a)-c^2(a+b)$

 

Giả sử $a=\text{min{a;b;c}}$, đặt $2t=b+c$

 

Khi đó $f(a;b;c)-f(a;t;t)=\left[b+c-\dfrac{5}{4}a \right](b-c)^2 \geqslant 0$

 

$\Leftrightarrow f(a;b;c) \geqslant f(a;t;t)=a^3+at^2-2a^2t=a(a^2-2at+t^2)=a(a-t)^2 \geqslant 0$




#567627 Topic tổng hợp một số bất đẳng thức trong kì thi MO các nước

Đã gửi bởi dogsteven on 23-06-2015 - 14:17 trong Bất đẳng thức - Cực trị

Sau đây là lời giải cho bài toán trên:

Không mất tính tổng quát ta giả sử a$\geq$b$\geq$c

Khi đó dễ thấy rằng:$a\geq b\geq c$ $\frac{1}{b^2+8bc+3c^2}\geq \frac{1}{c^2+8ca+3a^2}\geq \frac{1}{a^2+8ab+3b^2}$

Áp dụng bất đẳng thức Chebysev và bất đẳng thức AM-GM, ta có:

$\sum \frac{a}{b^2+4bc+3c^2}\geq \frac{1}{3}(a+b+c)(\sum \frac{1}{b^2+4bc+3c^2})\geq 3(a+b+c)(\frac{1}{4(a+b+c)^2})\geq \frac{1}{4}$ (đpcm)

Nên xem lại.




#567450 Topic tổng hợp một số bất đẳng thức trong kì thi MO các nước

Đã gửi bởi dogsteven on 22-06-2015 - 16:12 trong Bất đẳng thức - Cực trị

Dám hỏi dấu bằng xảy ra khi nào???

$a=b=1, c=0$ và các hoán vị.




#567215 Topic tổng hợp một số bất đẳng thức trong kì thi MO các nước

Đã gửi bởi dogsteven on 21-06-2015 - 07:03 trong Bất đẳng thức - Cực trị

Bài này thực sự là đặc chưng cho phương pháp tiếp tuyến, ta giải như sau:

1) Áp dụng bất đẳng thức AM-GM và giả thuyết thì chúng ta sẽ có BĐT đã cho tương đương BĐT sau:

$\sum \frac{1}{a^2-6a+13}\leq \frac{3}{8}$

Thật vậy lưu ý bất đẳng thức phụ sau:

$\frac{1}{a^2-6a+13}\leq \frac{a+1}{16}$

tương tự như thế suy ra$\sum \frac{1}{a^2-6a+13}\leq \frac{a+b+c+3}{16}=\frac{3}{8}$

từ đó ta có đpcm

Giả sử $a\geqslant b\geqslant c$. Bất đẳng thức trên tương đương với: $\sum \dfrac{(a+b)^2}{(a+b)^2+\dfrac{2(a+b)^2}{a^2+b^2}}\geqslant \dfrac{3}{2}$

Áp dụng bất đẳng thức Cauchy-Schwarz: $VT\geqslant \sum \dfrac{4(a+b+c)^2}{\sum (a+b)^2+\sum \dfrac{2(a+b)^2}{a^2+b^2}}$

Do đó ta cần chứng minh: $\sum (a+b)^2+\sum \dfrac{2(a+b)^2}{a^2+b^2}\leqslant 24$

$\Leftrightarrow \dfrac{4}{3} (a+b+c)^2-\sum (a+b)^2+12-\sum \dfrac{2(a+b)^2}{a^2+b^2}\geqslant 0\Leftrightarrow \sum (a-b)^2\left(\dfrac{6}{a^2+b^2}-1\right)\geqslant 0$

Nếu $a^2+b^2\leqslant 6$ thì bất đẳng thức đúng. Nếu $a^2+b^2\geqslant  6$ thì:

$\sum \dfrac{1}{a^2+b^2+2}\leqslant \dfrac{1}{8}+\dfrac{1}{a^2+2}+\dfrac{1}{b^2+6} \leqslant \dfrac{1}{8}+\dfrac{1}{8-b^2}+\dfrac{1}{b^2+2}\leqslant \dfrac{1}{8}+\dfrac{5}{8}=\dfrac{3}{4}$




#567628 Topic tổng hợp một số bất đẳng thức trong kì thi MO các nước

Đã gửi bởi dogsteven on 23-06-2015 - 14:20 trong Bất đẳng thức - Cực trị

Không biết bài này có chưa, nếu có rồi thì anh Đức xóa dùm em

Bài 142(APMO 1996) Cho a,b,c là độ dài 3 cạnh tam giác. Chứng minh rằng:

$\sqrt{a+b-c}+\sqrt{b+c-}+\sqrt{c+a-b}\geq \sqrt{a}+\sqrt{b}+\sqrt{c}$

Đẳng thức xảy ra khi nào

Bị ngược dấu rồi em.

Không mất tính tổng quát, giả sử $a\geslant b\geqslant c$

Dễ thấy $a+b-c\geqslant c+a-b\geqslant b+c-a$

Mà $a+b-c\geqslant a$ và $a+b-c+c+a-b=2a\geqslant a+b$ và $a+b-c+c+a-b+b+c-a=a+b+c$ nên $(a+b-c, c+a-b, b+c-a)\succ (a,b,c)$

Áp dụng bất đẳng thức Karamata cho hàm lõm $f(x)=\sqrt{x}$ cho ta điều phải chứng minh.




#568323 Topic tổng hợp một số bất đẳng thức trong kì thi MO các nước

Đã gửi bởi dogsteven on 26-06-2015 - 16:39 trong Bất đẳng thức - Cực trị

Bài 155:(Slovenia TST): Cho a,b,c>0. CMR: $a^2+b^2+c^2=3$ thì

$\sum \frac{a}{b+2}\leq 1$

Bất đẳng thức trên tương đương với: $ab^2+bc^2+ca^2-abc\leqslant 2$

Giả sử $a=\text{median}\{b,c\}$ thì $bc^2+ca^2-a(c^2+bc)=c(a-c)(a-b)\leqslant 0$ nên $ab^2+bc^2+ca^2-abc\leqslant a(b^2+c^2)=a(3-a^2)\leqslant \dfrac{[4+(a-1)^2]^2}{8}\leqslant 2$ do $0\leqslant a\leqslant \sqrt{3}$

Từ đó suy ra điều phải chứng minh. Đẳng thức xảy ra khi và chỉ khi $a=b=c=1$




#567137 Topic tổng hợp một số bất đẳng thức trong kì thi MO các nước

Đã gửi bởi dogsteven on 20-06-2015 - 17:00 trong Bất đẳng thức - Cực trị

Bài 130(IMO 2001): Cho a,b,c>0. CMR:

$\sum \frac{a}{\sqrt{a^2+8bc}}\geq 1$

Có thể dùng Holder nhưng không thích, ta dùng dao mổ trâu :v

$x=\sqrt{\dfrac{a^2}{a^2+8bc}}, y=..., z=...$, ta cần chứng minh $x+y+z\geqslant 1$

Chú ý rằng $(1-x^2)(1-y^2)(1-z^2)=8^3x^2y^2z^2$, nếu $x+y+z<1$ thì:

$8^3x^2y^2z^2>(y+z)(z+x)(x+y)(y+z+2x)(z+x+2y)(x+y+2z)$

Bất đẳng thức hiển nhiên sai theo AM-GM




#569456 Topic tổng hợp một số bất đẳng thức trong kì thi MO các nước

Đã gửi bởi dogsteven on 02-07-2015 - 15:23 trong Bất đẳng thức - Cực trị

Bài 167(China TST): Cho a,b,c thuộc [0;1]. Tìm GTNN của biểu thức sau:

                              P=$\frac{1}{(b-1)^2+a^2}+\frac{1}{(b-1)^2+c^2}$

Đề có thể sai chăng? Hiển nhiên $P\geqslant 1$, đẳng thức khi $a-1=b=c-1=0$

TST không thể dễ đến vậy




#569453 Topic tổng hợp một số bất đẳng thức trong kì thi MO các nước

Đã gửi bởi dogsteven on 02-07-2015 - 15:16 trong Bất đẳng thức - Cực trị

Bài 165 (Mediterranean Mathematical Competition 2009).Chứng minh rằng với mọi  $a,b,c $ dương ta luôn có:

$$\sum \frac{ab}{a^2+ab+b^2}\leq \frac{a}{2a+b}+\frac{b}{2b+c}+\frac{c}{2c+a}$$

Áp dụng bất đẳng thức Cauchy-Schwarz:

$3-\sum \dfrac{2b}{2b+a}=\sum \dfrac{a}{a+2b}\geqslant \dfrac{(a+b+c)^2}{a^2+b^2+c^2+2ab+2bc+2ca}=1\Rightarrow \sum \dfrac{b}{a+2b}\leqslant 1$

Do đó $\sum \dfrac{a}{2a+b}=\sum \dfrac{a}{2a+b}+\sum \dfrac{b}{2b+a}-1$

Ta sẽ chứng minh: $\dfrac{a}{2a+b}+\dfrac{b}{2b+a}\geqslant \dfrac{ab}{a^2+b^2+ab}+\dfrac{1}{3}$ hay $\dfrac{a^2+b^2+4ab}{(a+2b)(b+2a)}\geqslant \dfrac{a^2+b^2+4ab}{3(a^2+b^2+ab)}\Leftrightarrow (a-b)^2\geqslant 0$




#564471 Topic tổng hợp một số bất đẳng thức trong kì thi MO các nước

Đã gửi bởi dogsteven on 08-06-2015 - 20:11 trong Bất đẳng thức - Cực trị

Bài 59:(PP điểm rơi trong AM-GM)(VNTST 2001)

Cho các số thực dương $a,b,c$ thỏa mãn: $2x+4y+7z=2xyz$. Tìm giá trị nhỏ nhất của biểu thức: $P=x+y+z$

 

p/s: trình bày luôn cách tìm điểm rơi nhé!

Ta dùng nhân tử để tìm điểm rơi.

Xét hàm số $L=x+y+z+\lambda(2x+4y+7z-2xyz)$

Đạo hàm riêng cho biến $x$ được $1=2\lambda (yz-1)$

Đạo hàm riêng cho biến $y$ được $1=2\lambda(zx-2)$

Đạo hàm riêng cho biến $z$ được $1=\lambda(2xy-7)$

Do đó $2(yz-1)=2(zx-2)=2xy-7$




#560970 Topic tổng hợp một số bất đẳng thức trong kì thi MO các nước

Đã gửi bởi dogsteven on 22-05-2015 - 19:54 trong Bất đẳng thức - Cực trị

Bài 41. 

Giả sử $x=\text{min}\{x,y,z\}$, Thay trực tiếp $x=\dfrac{4-yz}{y+z+yz}$ ta được:

$VT-VP=y^2+z^2+\dfrac{(4-yz)^2}{y+z+yz}-2\left(1+\sqrt{2}\right)\left(y+z+\dfrac{4-yz}{y+z+yz}\right)+3+6\sqrt{2}$

Đặt $a=y+z$ và $b=yz$ thì ta có $f(b)=a^2-2b+\dfrac{(4-b)^2}{(a+b)^2}-2(1+\sqrt{2})\left(a+\dfrac{4-b}{a+b}\right)+3+6\sqrt{2}$

$=-2b+\dfrac{4-b}{(a+b)^2}\left[4-b-2(1+\sqrt{2})(a+b)\right]+a^2-2(1+\sqrt{2})a+3+6\sqrt{2}$

Có:

$4-b-2(1+\sqrt{2})(a+b)\geqslant 4-\dfrac{a^2}{4}-2(1+\sqrt{2})\left(a+\dfrac{a^2}{4}\right)$

$4-b\geqslant 4-\dfrac{a^2}{4}$

$-2b\geqslant \dfrac{a^2}{2}$

$\dfrac{1}{(a+b)^2}\geqslant \dfrac{1}{\left(a+\dfrac{a^2}{4}\right)^2}$

Do đó ta chỉ cần chứng minh khi $b=\dfrac{a^2}{4}$




#559479 Topic tổng hợp một số bất đẳng thức trong kì thi MO các nước

Đã gửi bởi dogsteven on 15-05-2015 - 10:04 trong Bất đẳng thức - Cực trị

Bài 6. $\sum a . \sum \dfrac{1}{a} - 9 =\sum \dfrac{(b-c)^2}{bc}\geqslant 3\sqrt[3]{\dfrac{\prod (b-c)^2}{(abc)^2}}$




#564469 Topic tổng hợp một số bất đẳng thức trong kì thi MO các nước

Đã gửi bởi dogsteven on 08-06-2015 - 20:07 trong Bất đẳng thức - Cực trị

 Bất đẳng thức đồng bậc 3 nên ta chuẩn hoá $a^2+b^2+c^2=9$

 Khi đó : BĐT $\Leftrightarrow 2(a+b+c)-abc\leq 10\Leftrightarrow a(2-bc)+(b+c).2\leq 10$

 Giả sử $a\geq b\geq c \Rightarrow |bc|\leq \frac{9-a^2}{2}\leq 3 \Rightarrow bc\in [-3;3]$

 Áp dụng Cauchy-Schwarz ta có : $a(2-bc)+(b+c).2\leq \sqrt{(9+2bc)(b^2c^2-4bc+8)}\leq 10\Leftrightarrow (bc+2)^2(2bc-7)\leq 0$

 Luôn đúng $\forall ~bc\in [-3;3]$

 Từ đó có đpcm

Ta có một lời giải rất xấu bằng phân tách trường hợp như sau:

Chuẩn hóa $a^2+b^2+c^2=9$. Giả sử $a\geqslant b\geqslant c$

Nếu $a\leqslant 0$ thì bất đẳng thức hiển nhiên đúng do $abc>-10$

Nếu $a\geqslant 0$ và $b,c\leqslant 0$ thì $2(a+b+c)\leqslant 2a\leqslant 6\leqslant 10\leqslant 10+abc$

Nếu $a\geqslant b\geqslant 0\geqslant c$ thì $2a+2b+2c\le 9+3c$ nên ta cần chứng minh $3c\le 1+\dfrac{c(9-c^2)}{2}$ luôn đúng.

Nếu $c\geqslant 0$ thì $2a+2b+2c\leqslant 9+c$

- Trường hợp $c\leqslant 1$ thì $9+c\leqslant 10\leqslant 10+abc$

- Trường hợp $c\geqslant 1$ thì $9+c\leqslant 9+abc\leqslant 10+abc$




#565745 Topic tổng hợp một số bất đẳng thức trong kì thi MO các nước

Đã gửi bởi dogsteven on 14-06-2015 - 20:58 trong Bất đẳng thức - Cực trị

 Nếu thay $a=1/3$ vào thì $k(a-1/3)=0$ như vậy là mất k rồi làm sao tìm được anh ?

Nhân tử lại đã. Rồi rút gọn cái $a-\dfrac{1}{3}$, sau đó mới cho $a=\dfrac{1}{3}$




#566839 Topic tổng hợp một số bất đẳng thức trong kì thi MO các nước

Đã gửi bởi dogsteven on 19-06-2015 - 13:00 trong Bất đẳng thức - Cực trị

Bài 115. 

Bất đẳng thức đã cho tương đương với: $\sum \left[\dfrac{1}{yz}-\dfrac{3}{(x+y)(x+z)}\right](y-z)^2\geqslant 0$

(1) $(y+z)(y+x)-3zx=y^2+y(z+x)-2zx=y(y+z)+xy-2zx\geqslant 2x(y-z)\geqslant 0$

(2) $\dfrac{1}{yz}-\dfrac{3}{(x+y)(x+z)}\geqslant \dfrac{1}{4yz}\geqslant 0$

(3) $\dfrac{1}{zx}+\dfrac{1}{xy}-\dfrac{3}{(y+z)(y+x)}-\dfrac{3}{(z+x)(z+y)}=\dfrac{x^2(y^2+z^2)+x(y^3+z^3)+yz(y+z)^2-4x^2yz}{xyz(x+y)(y+z)(z+x)}$

Mà $x^2(y^2+z^2)+x(y^3+z^3)+yz(y+z)^2-4x^2yz\geqslant x(y^3+z^3)-x^2yz\geqslant yz(y+z-x)\geqslant 0$

nên $\dfrac{1}{zx}+\dfrac{1}{xy}-\dfrac{3}{(y+z)(y+x)}-\dfrac{3}{(z+x)(z+y)}\geqslant 0$

Theo tiêu chuẩn S.O.S ta có điều phải chứng minh.




#565749 Topic tổng hợp một số bất đẳng thức trong kì thi MO các nước

Đã gửi bởi dogsteven on 14-06-2015 - 21:04 trong Bất đẳng thức - Cực trị

Tổng quát cách làm:

Với bất đẳng thức có dạng tổng hàm $f(x)+f(y)+f(z)\geqslant 0$ với điều kiện $x+y+z=3$ ta có thể sẽ chứng minh: $f(x)\geqslant f'(1)(x-1)+f(1)$

Với bất đẳng thức có dạng tổng hàm $f(x)+f(y)+f(z)\geqslant 0$ với điều kiện $xyz=1$ thì ta có thể sẽ chứng minh: $f(x)\geqslant f'(x)\ln x+f(1)$




#567136 Topic tổng hợp một số bất đẳng thức trong kì thi MO các nước

Đã gửi bởi dogsteven on 20-06-2015 - 16:56 trong Bất đẳng thức - Cực trị

Bài 131(Poland MO): Cho a,b,c>0. CMR

$\sqrt{\frac{a^5-a^2+3}{4b+5c}}+\sqrt{\frac{b^5-b^2+3}{4c+5a}}+\sqrt{\frac{c^5-c^2+3}{4a+5b}}\geq 1$

$\prod (a^5-a^3+3)\geqslant \prod (a^2+2)\geqslant (a+b+c)^3$ còn $\prod (4b+5c)\leqslant 3^3(a+b+c)^3$

Do đó $VT\geqslant \sqrt{3}>1$